Warum fällt dieses Modell auseinander, wenn die Winkelgeschwindigkeit klein ist?

Ich mache eine Physikaufgabe, bei der sich eine Murmel um eine sich drehende Schüssel dreht und beide eine Winkelgeschwindigkeit haben ω . Es dreht sich mit Radius R um die Mittelachse und die halbkugelförmige Schale hat einen Radius R . Ich habe für den Radius gelöst R bezüglich ω :

R = R 2 G 2 ω 4

Aber ich kann nicht herausfinden, was das wann bedeutet ω ist wirklich klein ( < G R ).

Einige Hypothesen:

  • Irgendwas mit Reibung
  • Es fällt durch die Mitte der Schüssel (wenn sie hohl wäre)
  • Wir vermissen etwas in diesem Modell

Sind einige davon richtig? Ist das komplexer als es scheint?

Hat jemand eine sich drehende Schüssel, eine Murmel und eine experimentierfreudige Denkweise parat? ;) [Im Ernst, warum bewegt sich die Murmel in dieser Anordnung überhaupt? Ich stelle mir eine Murmel vor, die sich in der Mitte einer Schüssel dreht, aber das ist offensichtlich nicht das, was Sie meinen.]
Wenn Sie Ihre Ableitung zeigen, wäre es einfacher zu kommentieren. Es kann keine Reibung sein – es gibt keine. Möglicherweise verwenden Sie eine Näherung für kleine Winkel. Wie es scheint R sollte problemlos gehen 0 mit ω
Wenn Sie den Ball langsam genug drehen, wird er sich schließlich nicht vom Boden der Schüssel lösen.
wie ist R verwandt sein R , dh R > R oder R < R ?
Dies scheint meiner älteren Frage hier sehr ähnlich zu sein: physical.stackexchange.com/questions/17513/…

Antworten (1)

Ich habe deine Berechnung nachgestellt. Wenn θ ist der Winkel von der vertikalen Mittelachse der Halbkugel zur Kugel, die tangential nach unten gerichtete Kraft ist G Sünde θ = G R R Die tangentiale Aufwärtskraft aufgrund der Rotation ist ω 2 R cos θ = ω 2 R 1 R 2 R 2 Wenn ω < G R Die nach unten gerichtete Kraft ist immer größer und die Kugel sitzt am Boden der Schüssel. Wenn ω > G R Sie bekommen die richtige Antwort.